¿Por qué no hay anomalía cuando se cuantifica la mecánica de partículas?

Sabemos que si se viola una o más simetrías de la acción de una teoría clásica de campos en su versión cuantizada, se dice que la teoría cuántica correspondiente tiene anomalía.

  1. ¿Es esta una característica única de la cuantización de una teoría de campo? En caso afirmativo, ¿por qué las anomalías aparecen solo después de cuantificar una teoría de campo pero no en la mecánica cuántica ordinaria no relativista?

En la teoría de campos, si bajo una transformación de simetría arbitraria ϕ ϕ = ϕ + d ϕ , la acción S [ ϕ ] se deja invariante, tenemos una simetría en la teoría clásica de campos. Pero tenemos una simetría de la teoría cuántica de campos, si la transformación deja la integral de trayectoria D ϕ Exp ( i S [ ϕ ] ) invariante. Por lo tanto, incluso si S ( ϕ ) es invariante pero la medida no lo es, podemos tener una anomalía.

  1. ¿Significa que la medida integral de trayectoria D q ( t ) Exp ( i S [ q ( t ) ] ) en la mecánica cuántica ordinaria siempre permanece invariante bajo cualquier simetría clásica q q = q + d q ?

Respuestas (3)

La mecánica cuántica también puede volverse anómala. Un ejemplo es una partícula cargada que se mueve en un campo magnético uniforme. En el nivel clásico, el sistema es invariante de traslación tanto en la dirección x como en la dirección y. Debido a que el campo magnético es uniforme, todas las mediciones (invariantes de calibre) arrojarán el mismo resultado en cualquier punto del espacio, por lo que se conserva la simetría de traslación. Pero una vez que se cuantifica el sistema, el impulso pag X y pag y ya no viajan entre sí, es decir

[ pag X , pag y ] = i B .
La no conmutatividad es exactamente proporcional a , lo que implica que este es de hecho un efecto cuántico. En este caso, si se elige conservar la traslación a lo largo de x, la traslación a lo largo de y debe romperse, como pag X y pag y convertirse en observables incompatibles. Este efecto se manifiesta en la función de onda bajo el gálibo de Landau. Por lo tanto, el sistema se vuelve anómalo bajo traducción.

Otro ejemplo estrechamente relacionado es una partícula cargada que se mueve sobre una esfera con un monopolo magnético (monopolo de Dirac) dentro de la esfera. Sea el vector unitario norte = ( norte 1 , norte 2 , norte 3 ) Sea la coordenada que parametriza la posición de la partícula sobre la esfera ( norte 2 = 1 ). La acción clásica se puede escribir como un modelo de Wess-Zumino-Witten

S [ norte ( t ) ] = 1 4 π d t 0 1 d tu norte t norte × tu norte .
La acción es invariante bajo la transformación SO(3) de norte . Pero después de la cuantificación, los estados propios son objetos de espín 1/2, que no son representaciones lineales del grupo de simetría SO(3). Entonces el sistema tiene una anomalía SO(3).

No entiendo tu primer ejemplo. Hay dos tipos de "momentum" en este ejemplo: el momento PAG i y momento cinemático pag i = metro v i = PAG i A i ( q ) . Los impulsos conmutan en el nivel clásico y cuántico, pero los impulsos cinemáticos (que usa) no conmutan ya en el nivel clásico, es decir, para el soporte de Poisson { pag X , pag y } = B . ¿Me estoy perdiendo de algo?
@Alex En mecánica cuántica, el conmutador que no desaparece [ pag X , pag y ] 0 implica que pag X y pag y no puede medirse de manera simultánea y consistente, debido a la relación de incertidumbre. En mecánica clásica, el soporte de Poisson que no desaparece { pag X , pag y } 0 , sin embargo, no tiene la misma consecuencia física (no existe una relación de incertidumbre en la mecánica clásica). Esta es una diferencia importante entre el soporte de Poisson y el conmutador. Es la relación de incertidumbre entre pag X y pag y que da lugar a la anomalía cuántica.

Las anomalías no son particulares de la teoría cuántica de campos, ni siquiera de la teoría cuántica. Una anomalía es una obstrucción para representar algún grupo/álgebra físicamente relevante, a menudo un grupo de simetría o un álgebra de observables, en el espacio de estado, y significa que su espacio de estado no llevará una representación del grupo de simetría en sí, sino una extensión. Esta noción se explica detalladamente en esta excelente respuesta de David Bar Moshe .

Si el grupo/álgebra que está obstruido es el grupo galileano clásico que necesita la introducción de la masa como una "carga central" para convertirse en el grupo de Bargmann, el S O ( 3 ) de una partícula como en la respuesta de Everett You (que es un caso especial de un vínculo más general entre modelos WZW y anomalías) que necesita el pasaje a su cobertura universal S tu ( 2 ) en el spin-1/2 que es una extensión central por Z 2 o el álgebra de densidades de carga fermiónica no abeliana que se extiende al álgebra de Mickelsson-Faddeev (ver nuevamente la respuesta de David) por el término de anomalía es irrelevante: todo es el mismo principio.

El carácter fundamental de una anomalía no es la no invariancia de la medida de la integral de trayectoria, es solo una forma particular de derivarla.

En referencia a la primera respuesta: Alex tenía razón. Este no es un efecto cuántico, ya que también surge en el límite clásico del corchete de Poisson cuantizado, que se define como el conmutador dividido por iħ. La forma clásica de "no conmuta" es que "los campos hamiltonianos asociados con las cantidades en cuestión no conmutan". Aquí, los campos hamiltonianos son X = {p,⋯}, y lo que encontramos es que

    [X₁,X₂] = X₁X₂ - X₂X₁ = {p₁,{p₂,⋯}} - {p₂,{p₁,⋯}} = {{p₁,p₂},⋯} = {B³,⋯} ≠ 0.

El criterio de "los campos hamilonianos no conmutan" es también la forma cuántica de "no conmutan", ya que los corchetes de Poisson cuantificados son, de hecho, corchetes de Poisson sobre una variedad de Poisson adecuadamente definida, y sus campos hamiltonianos proporcionan descripciones equivalentes de los operadores , ellos mismos. Por lo tanto, la condición se puede usar de manera uniforme en todos los ámbitos, independientemente de la división entre el paradigma clásico y el cuántico.

Un "efecto cuántico" es lo que surge como una corrección cuántica, por ejemplo, Q({p₁,p₂}) frente a {Q(p₁),Q(p₂)}, donde el segundo paréntesis es el paréntesis de Poisson cuantificado; y donde Q() denota la operación de cuantificación.

Esto sigue el tratamiento dado en Landsman ( Mathematical Topics Between Classical and Quantum Mechanics (1998), Part II "Quantization and the Classical Limit"). Aquí, la diferencia Q({p₁,p₂}) - {Q(p₁),Q(p₂)} = Q(B³) - Q(B³) es 0. Discusión de los espacios proyectivos de Hilbert como variedades de Poisson (en realidad, variedades simplécticas) se puede encontrar en la sección I.2 en Landsman.

Las anomalías son déficits que surgen de un intento de cuantificar una transformada de simetría clásica, que estropea la transformada de simetría a nivel cuántico. Varios ejemplos, relacionados con la pregunta original, se ilustran aquí https://www.physics.ncsu.edu/ntg/leegroup/library/Anomaly_QM.pdf y la discusión de anomalías, en mayor profundidad, se puede encontrar aquí https:// en.wikipedia.org/wiki/Anomaly_%28physics%29